Why do we call complex numbers “numbers” but we don’t consider 2-vectors numbers?What exactly is a number?Examples of bijective map from $mathbbR^3rightarrow mathbbR$Why are complex numbers considered to be numbers?Relationship between complex number and vectorsAre there any numbers more fundamental than Complex numbers?Why are complex numbers considered to be numbers?Does it make sense to compare complex numbers in certain circumstances?Usefulness of alternative constructions of the complex numbersComplex numbers?Why do we call $sqrt-1$ imaginary and $-1$ real?What can complex numbers do that linear algebra cannot?complex numbers and rotation matricesIf there is anything else introduced into equations like the complex numbers.

Quoting Keynes in a lecture

Will number of steps recorded on FitBit/any fitness tracker add up distance in PokemonGo?

Did the UK lift the requirement for registering SIM cards?

Taxes on Dividends in a Roth IRA

Non-trope happy ending?

What is the highest possible scrabble score for placing a single tile

How much theory knowledge is actually used while playing?

Biological Blimps: Propulsion

Does "he squandered his car on drink" sound natural?

How do you make your own symbol when Detexify fails?

What's the name of the logical fallacy where a debater extends a statement far beyond the original statement to make it true?

What is the difference between lands and mana?

Why should universal income be universal?

What features enable the Su-25 Frogfoot to operate with such a wide variety of fuels?

Does an advisor owe his/her student anything? Will an advisor keep a PhD student only out of pity?

How do I fix the group tension caused by my character stealing and possibly killing without provocation?

Why is it that I can sometimes guess the next note?

Does the reader need to like the PoV character?

Can I say "fingers" when referring to toes?

How to convince somebody that he is fit for something else, but not this job?

Pre-mixing cryogenic fuels and using only one fuel tank

How many arrows is an archer expected to fire by the end of the Tyranny of Dragons pair of adventures?

A variation to the phrase "hanging over my shoulders"

How to get directions in deep space?



Why do we call complex numbers “numbers” but we don’t consider 2-vectors numbers?


What exactly is a number?Examples of bijective map from $mathbbR^3rightarrow mathbbR$Why are complex numbers considered to be numbers?Relationship between complex number and vectorsAre there any numbers more fundamental than Complex numbers?Why are complex numbers considered to be numbers?Does it make sense to compare complex numbers in certain circumstances?Usefulness of alternative constructions of the complex numbersComplex numbers?Why do we call $sqrt-1$ imaginary and $-1$ real?What can complex numbers do that linear algebra cannot?complex numbers and rotation matricesIf there is anything else introduced into equations like the complex numbers.













30












$begingroup$


We refer to complex numbers as numbers. However we refer to vectors as arrays of numbers. There doesn’t seem to be anything that makes one more numeric than the other. Is this just a quirk of history and naming or is there something more fundamental?










share|cite|improve this question











$endgroup$







  • 11




    $begingroup$
    Usually, the difference would be whether you're in a context where complex multiplication would be interesting. It's kind of like the question: is $mathbbZ$ a group or a ring? It depends on which operations are of interest for the question you're studying.
    $endgroup$
    – Nate Eldredge
    Mar 7 at 23:20







  • 14




    $begingroup$
    What's a "number" anyway?
    $endgroup$
    – Asaf Karagila
    Mar 7 at 23:33






  • 13




    $begingroup$
    @QthePlatypus: No, that's exactly the point. The question of whether the set $mathbbR^2$ of pairs of real numbers should be considered as the set of complex numbers, or as a set of vectors, depends on which operations you wish to equip that set with.
    $endgroup$
    – Nate Eldredge
    Mar 7 at 23:48






  • 8




    $begingroup$
    I think the correct question were: is there a rigorous definition of the object "number" (without further characteristics such as "integer", "real", "complex" etc.)?
    $endgroup$
    – user
    Mar 7 at 23:57







  • 1




    $begingroup$
    Some considerations can be found here: math.stackexchange.com/q/865409
    $endgroup$
    – user
    Mar 8 at 0:01
















30












$begingroup$


We refer to complex numbers as numbers. However we refer to vectors as arrays of numbers. There doesn’t seem to be anything that makes one more numeric than the other. Is this just a quirk of history and naming or is there something more fundamental?










share|cite|improve this question











$endgroup$







  • 11




    $begingroup$
    Usually, the difference would be whether you're in a context where complex multiplication would be interesting. It's kind of like the question: is $mathbbZ$ a group or a ring? It depends on which operations are of interest for the question you're studying.
    $endgroup$
    – Nate Eldredge
    Mar 7 at 23:20







  • 14




    $begingroup$
    What's a "number" anyway?
    $endgroup$
    – Asaf Karagila
    Mar 7 at 23:33






  • 13




    $begingroup$
    @QthePlatypus: No, that's exactly the point. The question of whether the set $mathbbR^2$ of pairs of real numbers should be considered as the set of complex numbers, or as a set of vectors, depends on which operations you wish to equip that set with.
    $endgroup$
    – Nate Eldredge
    Mar 7 at 23:48






  • 8




    $begingroup$
    I think the correct question were: is there a rigorous definition of the object "number" (without further characteristics such as "integer", "real", "complex" etc.)?
    $endgroup$
    – user
    Mar 7 at 23:57







  • 1




    $begingroup$
    Some considerations can be found here: math.stackexchange.com/q/865409
    $endgroup$
    – user
    Mar 8 at 0:01














30












30








30


3



$begingroup$


We refer to complex numbers as numbers. However we refer to vectors as arrays of numbers. There doesn’t seem to be anything that makes one more numeric than the other. Is this just a quirk of history and naming or is there something more fundamental?










share|cite|improve this question











$endgroup$




We refer to complex numbers as numbers. However we refer to vectors as arrays of numbers. There doesn’t seem to be anything that makes one more numeric than the other. Is this just a quirk of history and naming or is there something more fundamental?







matrices complex-numbers terminology philosophy






share|cite|improve this question















share|cite|improve this question













share|cite|improve this question




share|cite|improve this question








edited Mar 8 at 11:05









Basj

4071529




4071529










asked Mar 7 at 23:15









Q the PlatypusQ the Platypus

2,9121135




2,9121135







  • 11




    $begingroup$
    Usually, the difference would be whether you're in a context where complex multiplication would be interesting. It's kind of like the question: is $mathbbZ$ a group or a ring? It depends on which operations are of interest for the question you're studying.
    $endgroup$
    – Nate Eldredge
    Mar 7 at 23:20







  • 14




    $begingroup$
    What's a "number" anyway?
    $endgroup$
    – Asaf Karagila
    Mar 7 at 23:33






  • 13




    $begingroup$
    @QthePlatypus: No, that's exactly the point. The question of whether the set $mathbbR^2$ of pairs of real numbers should be considered as the set of complex numbers, or as a set of vectors, depends on which operations you wish to equip that set with.
    $endgroup$
    – Nate Eldredge
    Mar 7 at 23:48






  • 8




    $begingroup$
    I think the correct question were: is there a rigorous definition of the object "number" (without further characteristics such as "integer", "real", "complex" etc.)?
    $endgroup$
    – user
    Mar 7 at 23:57







  • 1




    $begingroup$
    Some considerations can be found here: math.stackexchange.com/q/865409
    $endgroup$
    – user
    Mar 8 at 0:01













  • 11




    $begingroup$
    Usually, the difference would be whether you're in a context where complex multiplication would be interesting. It's kind of like the question: is $mathbbZ$ a group or a ring? It depends on which operations are of interest for the question you're studying.
    $endgroup$
    – Nate Eldredge
    Mar 7 at 23:20







  • 14




    $begingroup$
    What's a "number" anyway?
    $endgroup$
    – Asaf Karagila
    Mar 7 at 23:33






  • 13




    $begingroup$
    @QthePlatypus: No, that's exactly the point. The question of whether the set $mathbbR^2$ of pairs of real numbers should be considered as the set of complex numbers, or as a set of vectors, depends on which operations you wish to equip that set with.
    $endgroup$
    – Nate Eldredge
    Mar 7 at 23:48






  • 8




    $begingroup$
    I think the correct question were: is there a rigorous definition of the object "number" (without further characteristics such as "integer", "real", "complex" etc.)?
    $endgroup$
    – user
    Mar 7 at 23:57







  • 1




    $begingroup$
    Some considerations can be found here: math.stackexchange.com/q/865409
    $endgroup$
    – user
    Mar 8 at 0:01








11




11




$begingroup$
Usually, the difference would be whether you're in a context where complex multiplication would be interesting. It's kind of like the question: is $mathbbZ$ a group or a ring? It depends on which operations are of interest for the question you're studying.
$endgroup$
– Nate Eldredge
Mar 7 at 23:20





$begingroup$
Usually, the difference would be whether you're in a context where complex multiplication would be interesting. It's kind of like the question: is $mathbbZ$ a group or a ring? It depends on which operations are of interest for the question you're studying.
$endgroup$
– Nate Eldredge
Mar 7 at 23:20





14




14




$begingroup$
What's a "number" anyway?
$endgroup$
– Asaf Karagila
Mar 7 at 23:33




$begingroup$
What's a "number" anyway?
$endgroup$
– Asaf Karagila
Mar 7 at 23:33




13




13




$begingroup$
@QthePlatypus: No, that's exactly the point. The question of whether the set $mathbbR^2$ of pairs of real numbers should be considered as the set of complex numbers, or as a set of vectors, depends on which operations you wish to equip that set with.
$endgroup$
– Nate Eldredge
Mar 7 at 23:48




$begingroup$
@QthePlatypus: No, that's exactly the point. The question of whether the set $mathbbR^2$ of pairs of real numbers should be considered as the set of complex numbers, or as a set of vectors, depends on which operations you wish to equip that set with.
$endgroup$
– Nate Eldredge
Mar 7 at 23:48




8




8




$begingroup$
I think the correct question were: is there a rigorous definition of the object "number" (without further characteristics such as "integer", "real", "complex" etc.)?
$endgroup$
– user
Mar 7 at 23:57





$begingroup$
I think the correct question were: is there a rigorous definition of the object "number" (without further characteristics such as "integer", "real", "complex" etc.)?
$endgroup$
– user
Mar 7 at 23:57





1




1




$begingroup$
Some considerations can be found here: math.stackexchange.com/q/865409
$endgroup$
– user
Mar 8 at 0:01





$begingroup$
Some considerations can be found here: math.stackexchange.com/q/865409
$endgroup$
– user
Mar 8 at 0:01











8 Answers
8






active

oldest

votes


















45












$begingroup$

They're called "numbers" for historical reasons, since the motivation in the development of the complex numbers was solving polynomial equations. They were viewed as natural extensions of the real numbers. It's somehow quite natural and satisfying to say "every polynomial equation can be solved by some (complex) number". Is it more natural to regard $i$ as being a number which, when squared, is equal to $-1$, or is it more natural to regard $i$ as being some non-number thingamajig which when squared is equal to $-1$? Clearly the former.



"Higher" number systems, like quaternions, aren't really called numbers very often, for the simple fact that they are not as intimately connected with number theory and analysis in the same way that complex numbers are.



Beyond these social conventions, I can't see any other reason. The word "number" doesn't have a strict or absolute definition in pure mathematics. $mathbbN$, $mathbbZ$, $mathbbQ$, $mathbbR$ and $mathbbC$ are technically just sets with a certain algebraic structure.



I partially disagree with the other answers which claim that complex numbers are numbers simply by virtue of the fact that you can add and multiply them. Well, if that's the rationale, is every ring also a set of numbers?






share|cite|improve this answer











$endgroup$








  • 7




    $begingroup$
    Wikipedia calls the quaternions a number system.
    $endgroup$
    – JJJ
    Mar 8 at 4:52






  • 6




    $begingroup$
    You can also tell what opinion people had of a number system based on its name, generally (it is not a coincidence that the word rational means both a number system and a way of thinking). At least the old ones.
    $endgroup$
    – PyRulez
    Mar 8 at 6:26






  • 1




    $begingroup$
    Agreed; the use of the word number is definitely a matter of convention. As for opinions; the use of the word 'real' numbers is regarded as an incredibly triggering piece of propaganda by physical finitists...
    $endgroup$
    – Eelco Hoogendoorn
    Mar 8 at 12:40






  • 2




    $begingroup$
    @PyRulez I would have expected that "rational" comes from "ratio", not from "ration".
    $endgroup$
    – Thern
    Mar 8 at 16:31






  • 2




    $begingroup$
    @Thern The English words "ratio", "rational" and "reason" all come from the same Latin root "ratio", meaning "reckoning" or "reasoning"
    $endgroup$
    – David Richerby
    Mar 8 at 20:12


















20












$begingroup$

The two fundamental operations for numbers are "addition" and "multiplication" which obey very nice "laws" of arithmetic. Taking powers is also important. You can do all of those things with complex numbers. You can add two vectors but the "dot" product of two vectors is not a vector and the "cross" product of two vectors does not satisfy the "nice laws". Neither the dot product nor the cross product of vectors can be used to define powers.






share|cite|improve this answer









$endgroup$












  • $begingroup$
    So Quaternions which have Non commutativity multiplication would also be considered as non numbers?
    $endgroup$
    – Q the Platypus
    Mar 7 at 23:34






  • 5




    $begingroup$
    That's a good point- though I have never liked quaternions! Multiplication of quaternions is not commutative but it is associative which allows powers. That's what I was really thinking of.
    $endgroup$
    – user247327
    Mar 7 at 23:42






  • 4




    $begingroup$
    You can also add and multiply matrices, but I don't think I've ever heard anyone call a matrix a "number".
    $endgroup$
    – Hong Ooi
    Mar 8 at 2:48






  • 8




    $begingroup$
    @user247327: It's worth pointing out that IEEE 754 floating point numbers are non-associative, closed under division, non-reflexive (i.e. they violate $forall a:a = a$), non-substitutable (i.e. numbers which are "equal" cannot always be substituted for one another in expressions or equations), and also the most widely used approximation of the reals in practical computing applications.
    $endgroup$
    – Kevin
    Mar 8 at 2:48







  • 4




    $begingroup$
    @Todd Sewell - I think this may relate to positive and negative zero. Positive and negative zero are equal, yet if you divide something by one or the other you may get positive or negative infinity.
    $endgroup$
    – Hammerite
    Mar 8 at 13:27


















4












$begingroup$

Numbers appeared first when we started to count things. $1$ tree, $2$ trees, $3$ trees, and so forth. That make up the set of non-zero natural numbers: $mathbbN$. Afterwards, people started to "count backwards" to get $mathbbZ$ (I'm just kidding, you can do some research on how negative numbers appeared historically). With the urge to divide things without remainders, the set of rational numbers $mathbbQ$ made its way to the world. A certain idea of geometric continuity gives us $mathbbR$. Finally we want all equations to have a root, that's how $mathbbC$ comes into play.



I guess what is considered "numbers" is rather a social question. The use of $mathbbC$
in physics and its $2$-D representation must have given a good intuition for a large set of people to accept it's intuitive enough to be considered "numbers".



I think it's not that natural to think of $mathbbC$ as a $mathbbR$-vector space of dimension $2$, not more natural than to think of $mathbbR$ as an infinite-dimensional $mathbbQ$-vector space, nor of $mathbbQ$ as a non-infinitely-generated $mathbbZ$-module.






share|cite|improve this answer











$endgroup$








  • 3




    $begingroup$
    People do think of complex numbers as points on the number plane.
    $endgroup$
    – Q the Platypus
    Mar 7 at 23:48






  • 1




    $begingroup$
    Thank you. I'm sorry, that final part is just a personal opinion. edited.
    $endgroup$
    – Leaning
    Mar 7 at 23:48



















3












$begingroup$

I think naming is a rather soft topic, so there may not be a hard answer. However, I think it is worth noting that complex numbers are one of the three associative real division algebras (real numbers, complex numbers, and quaternions). These are all linked by the idea that division is meaningful in those three systems.



In general, a 2 dimensional real vector cannot admit a concept of division which matches what we expect division to do. However, if said real vector defines addition and multiplication the way complex numbers do, division is a natural result of those definitions.






share|cite|improve this answer









$endgroup$




















    2












    $begingroup$

    Both, complex numbers and vector spaces are mathematical spaces, i.e., sets equipped it with some structure, e.g., operations (such as addition or multiplication) and rules obeyed by them (such as commutativity or associativity).
    Without that structure, these sets are pretty useless:
    You can hardly make any interesting statements about them or connect them to reality.



    For example, without any structure, the only difference between ℝ and ℝ² is how we tend to name the objects within: We can find bijective maps between them.
    Of course, once we impose the usual structure onto these sets, we will note that these maps do not preserve any of it (they are not even continuous) and thus are not very relevant.



    Now, for ℝ² (the space of 2-vectors) and ℂ, some structure such as addition is the same, but some isn’t, e.g., there is no equivalent to complex multiplication in ℝ².
    This distinction in structure is the only relevant difference between ℝ² and ℂ.
    If you define a multiplication of 2-vectors that just mirrors complex multiplication, you would just be inventing complex numbers under a different name.



    So much for why it is justified to apply different labels to complex numbers and 2-vectors.
    As for why we call complex numbers numbers in the first place, there’s certainly a bit of history involved (of which I know little) and number is not a mathematically defined concept anyway.
    That being said, calling complex numbers numbers is not completely deviod of consistency, given that complex numbers feature most of the structural properties of other sets that we call numbers (for even more historical reasons), e.g., the vast majority of things you can do with real numbers, you can also do with complex numbers.
    This does not hold for 2-vectors.






    share|cite|improve this answer









    $endgroup$




















      1












      $begingroup$

      The answer by @MathematicsStudent1122 is good but doesn't address the question about vectors. The reason for using a distinctive name for such objects is that, in general, they do not admit all the arithmetical operations that have we come to expect with numbers. In particular, there is generally no natural or canonical way of multiplying or dividing vectors. Exceptionally, for two, three, or four dimensions, operations that correspond to multiplication or division of vectors do exist, although they are no longer commutative for dimensions 3 or 4; moreover, in three dimensions, vector multiplication has un-numberlike properties (e.g. $pmb xtimespmb x=pmb0$), and there is no natural way to define division. As you point out, real 2-D vectors can be identified with complex numbers. However, even in low-dimensional cases, typically vectors are not used in a context where any vector multiplication that could be defined has meaning.






      share|cite|improve this answer









      $endgroup$




















        1












        $begingroup$

        In my observation, the term number is used almost exclusively in the following cases:



        • The natural numbers, basically the origin of the term.



        • Any system which extends a system already called “numbers” by adding numbers that are in some sense “missing” That is, there are some constructions which sometimes give a number, but sometimes don't, although is “looks like” such a number should exist.



          The actual construction of the extension is then different from just adding those “missing” numbers just in order to make sure that what you do makes sense. However the goal is always to add the “missing” numbers, and any further numbers that are implied by their existence, but nothing else.



        The classic sequence of number sets is exactly of this type:



        • The natural numbers allow to solve equations like $2+x=6$, but not equations like $5+x=2$. Adding solutions to the latter gives the integers (which, of course, are considered to be numbers).


        • The integers allow to solve equations like $2x=6$, but not equations like $5x=2$. Adding solutions to the latter gives the rational numbers.


        • In the rational numbers, all convergent sequences are Cauchy, but not all Cauchy sequences converge. But they look like they should converge. Thus the missing limits are added. (Note that there are other constructions of the real numbers, based on other constructions where numbers are found "missing").


        • In the real numbers, equations like $x^2=2$ can be solved, but equations like $x^2=-2$ cannot. Adding those numbers then gives the complex numbers.


        But is also is true for most other systems which are generally called numbers. For example:



        • Finite sets always have an integer number of elements. For infinite sets, there's no natural number describing the set's size. The cardinal numbers add those numbers describing the size of infinite sets.


        • Similarly, when considering well-orderings, you need a generalization of the terms “first”, “second”, … to infinite size orderings. The ordinal numbers provide those missing numbers.


        • When informally talking about differentiation and integration, one often uses the concept of “infinitesimal quantities”. Those don't exist in the real or complex numbers. Non-standard analysis adds those “missing” infinitesimals (and everything implied by them), arriving at the hyperreal numbers.


        • When looking at the sequence $r_n$ of remainders modulo $p^n$, $p$ prime, every integer gives an unique sequence. But not every sequence gives a number. The $p$-adic numbers add those “missing” numbers.


        Indeed, off the top of my head, I can only think of two constructions called “numbers” that fall outside this scheme:



        • The quaternions are not constructed as extension, but explicitly as “numbers” that allow to describe three-dimensional geometry, after seeing that complex numbers describe two-dimensional geometry. It turned out that you need a fourth dimension to get something meaningful. However not everyone considers the quaternions to be numbers.


        • The surreal numbers are not constructed as extension to anything, but arise independently from combinatoric game theory. However since not only behave very much like numbers do, but in addition contain subsets isomorphic to several other number sets or classes (you'll find subsets isomorphic to the real numbers, to all systems of hyperreal numbers, and to the ordinal numbers under natural arithmetic). So while the surreal numbers are not constructed as extension, they can be viewed in some sense as such, justifying the term “number”.






        share|cite|improve this answer











        $endgroup$




















          0












          $begingroup$

          [Possibly not a direct answer, but]



          Notice ordinal sum and product are not commutative, that full distributivity of multiplication over addition and of exponentiation over product fails, and they're still regarded as numbers. OTOH, the corresponding operations for vector spaces are well-behaved (probably because they do become cardinal-like once you identify the isomorphic ones) and yet we don't tend to think of or to refer to them as numbers.






          share|cite|improve this answer









          $endgroup$












            Your Answer





            StackExchange.ifUsing("editor", function ()
            return StackExchange.using("mathjaxEditing", function ()
            StackExchange.MarkdownEditor.creationCallbacks.add(function (editor, postfix)
            StackExchange.mathjaxEditing.prepareWmdForMathJax(editor, postfix, [["$", "$"], ["\\(","\\)"]]);
            );
            );
            , "mathjax-editing");

            StackExchange.ready(function()
            var channelOptions =
            tags: "".split(" "),
            id: "69"
            ;
            initTagRenderer("".split(" "), "".split(" "), channelOptions);

            StackExchange.using("externalEditor", function()
            // Have to fire editor after snippets, if snippets enabled
            if (StackExchange.settings.snippets.snippetsEnabled)
            StackExchange.using("snippets", function()
            createEditor();
            );

            else
            createEditor();

            );

            function createEditor()
            StackExchange.prepareEditor(
            heartbeatType: 'answer',
            autoActivateHeartbeat: false,
            convertImagesToLinks: true,
            noModals: true,
            showLowRepImageUploadWarning: true,
            reputationToPostImages: 10,
            bindNavPrevention: true,
            postfix: "",
            imageUploader:
            brandingHtml: "Powered by u003ca class="icon-imgur-white" href="https://imgur.com/"u003eu003c/au003e",
            contentPolicyHtml: "User contributions licensed under u003ca href="https://creativecommons.org/licenses/by-sa/3.0/"u003ecc by-sa 3.0 with attribution requiredu003c/au003e u003ca href="https://stackoverflow.com/legal/content-policy"u003e(content policy)u003c/au003e",
            allowUrls: true
            ,
            noCode: true, onDemand: true,
            discardSelector: ".discard-answer"
            ,immediatelyShowMarkdownHelp:true
            );



            );













            draft saved

            draft discarded


















            StackExchange.ready(
            function ()
            StackExchange.openid.initPostLogin('.new-post-login', 'https%3a%2f%2fmath.stackexchange.com%2fquestions%2f3139486%2fwhy-do-we-call-complex-numbers-numbers-but-we-don-t-consider-2-vectors-numbers%23new-answer', 'question_page');

            );

            Post as a guest















            Required, but never shown

























            8 Answers
            8






            active

            oldest

            votes








            8 Answers
            8






            active

            oldest

            votes









            active

            oldest

            votes






            active

            oldest

            votes









            45












            $begingroup$

            They're called "numbers" for historical reasons, since the motivation in the development of the complex numbers was solving polynomial equations. They were viewed as natural extensions of the real numbers. It's somehow quite natural and satisfying to say "every polynomial equation can be solved by some (complex) number". Is it more natural to regard $i$ as being a number which, when squared, is equal to $-1$, or is it more natural to regard $i$ as being some non-number thingamajig which when squared is equal to $-1$? Clearly the former.



            "Higher" number systems, like quaternions, aren't really called numbers very often, for the simple fact that they are not as intimately connected with number theory and analysis in the same way that complex numbers are.



            Beyond these social conventions, I can't see any other reason. The word "number" doesn't have a strict or absolute definition in pure mathematics. $mathbbN$, $mathbbZ$, $mathbbQ$, $mathbbR$ and $mathbbC$ are technically just sets with a certain algebraic structure.



            I partially disagree with the other answers which claim that complex numbers are numbers simply by virtue of the fact that you can add and multiply them. Well, if that's the rationale, is every ring also a set of numbers?






            share|cite|improve this answer











            $endgroup$








            • 7




              $begingroup$
              Wikipedia calls the quaternions a number system.
              $endgroup$
              – JJJ
              Mar 8 at 4:52






            • 6




              $begingroup$
              You can also tell what opinion people had of a number system based on its name, generally (it is not a coincidence that the word rational means both a number system and a way of thinking). At least the old ones.
              $endgroup$
              – PyRulez
              Mar 8 at 6:26






            • 1




              $begingroup$
              Agreed; the use of the word number is definitely a matter of convention. As for opinions; the use of the word 'real' numbers is regarded as an incredibly triggering piece of propaganda by physical finitists...
              $endgroup$
              – Eelco Hoogendoorn
              Mar 8 at 12:40






            • 2




              $begingroup$
              @PyRulez I would have expected that "rational" comes from "ratio", not from "ration".
              $endgroup$
              – Thern
              Mar 8 at 16:31






            • 2




              $begingroup$
              @Thern The English words "ratio", "rational" and "reason" all come from the same Latin root "ratio", meaning "reckoning" or "reasoning"
              $endgroup$
              – David Richerby
              Mar 8 at 20:12















            45












            $begingroup$

            They're called "numbers" for historical reasons, since the motivation in the development of the complex numbers was solving polynomial equations. They were viewed as natural extensions of the real numbers. It's somehow quite natural and satisfying to say "every polynomial equation can be solved by some (complex) number". Is it more natural to regard $i$ as being a number which, when squared, is equal to $-1$, or is it more natural to regard $i$ as being some non-number thingamajig which when squared is equal to $-1$? Clearly the former.



            "Higher" number systems, like quaternions, aren't really called numbers very often, for the simple fact that they are not as intimately connected with number theory and analysis in the same way that complex numbers are.



            Beyond these social conventions, I can't see any other reason. The word "number" doesn't have a strict or absolute definition in pure mathematics. $mathbbN$, $mathbbZ$, $mathbbQ$, $mathbbR$ and $mathbbC$ are technically just sets with a certain algebraic structure.



            I partially disagree with the other answers which claim that complex numbers are numbers simply by virtue of the fact that you can add and multiply them. Well, if that's the rationale, is every ring also a set of numbers?






            share|cite|improve this answer











            $endgroup$








            • 7




              $begingroup$
              Wikipedia calls the quaternions a number system.
              $endgroup$
              – JJJ
              Mar 8 at 4:52






            • 6




              $begingroup$
              You can also tell what opinion people had of a number system based on its name, generally (it is not a coincidence that the word rational means both a number system and a way of thinking). At least the old ones.
              $endgroup$
              – PyRulez
              Mar 8 at 6:26






            • 1




              $begingroup$
              Agreed; the use of the word number is definitely a matter of convention. As for opinions; the use of the word 'real' numbers is regarded as an incredibly triggering piece of propaganda by physical finitists...
              $endgroup$
              – Eelco Hoogendoorn
              Mar 8 at 12:40






            • 2




              $begingroup$
              @PyRulez I would have expected that "rational" comes from "ratio", not from "ration".
              $endgroup$
              – Thern
              Mar 8 at 16:31






            • 2




              $begingroup$
              @Thern The English words "ratio", "rational" and "reason" all come from the same Latin root "ratio", meaning "reckoning" or "reasoning"
              $endgroup$
              – David Richerby
              Mar 8 at 20:12













            45












            45








            45





            $begingroup$

            They're called "numbers" for historical reasons, since the motivation in the development of the complex numbers was solving polynomial equations. They were viewed as natural extensions of the real numbers. It's somehow quite natural and satisfying to say "every polynomial equation can be solved by some (complex) number". Is it more natural to regard $i$ as being a number which, when squared, is equal to $-1$, or is it more natural to regard $i$ as being some non-number thingamajig which when squared is equal to $-1$? Clearly the former.



            "Higher" number systems, like quaternions, aren't really called numbers very often, for the simple fact that they are not as intimately connected with number theory and analysis in the same way that complex numbers are.



            Beyond these social conventions, I can't see any other reason. The word "number" doesn't have a strict or absolute definition in pure mathematics. $mathbbN$, $mathbbZ$, $mathbbQ$, $mathbbR$ and $mathbbC$ are technically just sets with a certain algebraic structure.



            I partially disagree with the other answers which claim that complex numbers are numbers simply by virtue of the fact that you can add and multiply them. Well, if that's the rationale, is every ring also a set of numbers?






            share|cite|improve this answer











            $endgroup$



            They're called "numbers" for historical reasons, since the motivation in the development of the complex numbers was solving polynomial equations. They were viewed as natural extensions of the real numbers. It's somehow quite natural and satisfying to say "every polynomial equation can be solved by some (complex) number". Is it more natural to regard $i$ as being a number which, when squared, is equal to $-1$, or is it more natural to regard $i$ as being some non-number thingamajig which when squared is equal to $-1$? Clearly the former.



            "Higher" number systems, like quaternions, aren't really called numbers very often, for the simple fact that they are not as intimately connected with number theory and analysis in the same way that complex numbers are.



            Beyond these social conventions, I can't see any other reason. The word "number" doesn't have a strict or absolute definition in pure mathematics. $mathbbN$, $mathbbZ$, $mathbbQ$, $mathbbR$ and $mathbbC$ are technically just sets with a certain algebraic structure.



            I partially disagree with the other answers which claim that complex numbers are numbers simply by virtue of the fact that you can add and multiply them. Well, if that's the rationale, is every ring also a set of numbers?







            share|cite|improve this answer














            share|cite|improve this answer



            share|cite|improve this answer








            edited Mar 8 at 3:10

























            answered Mar 7 at 23:54









            MathematicsStudent1122MathematicsStudent1122

            8,97122668




            8,97122668







            • 7




              $begingroup$
              Wikipedia calls the quaternions a number system.
              $endgroup$
              – JJJ
              Mar 8 at 4:52






            • 6




              $begingroup$
              You can also tell what opinion people had of a number system based on its name, generally (it is not a coincidence that the word rational means both a number system and a way of thinking). At least the old ones.
              $endgroup$
              – PyRulez
              Mar 8 at 6:26






            • 1




              $begingroup$
              Agreed; the use of the word number is definitely a matter of convention. As for opinions; the use of the word 'real' numbers is regarded as an incredibly triggering piece of propaganda by physical finitists...
              $endgroup$
              – Eelco Hoogendoorn
              Mar 8 at 12:40






            • 2




              $begingroup$
              @PyRulez I would have expected that "rational" comes from "ratio", not from "ration".
              $endgroup$
              – Thern
              Mar 8 at 16:31






            • 2




              $begingroup$
              @Thern The English words "ratio", "rational" and "reason" all come from the same Latin root "ratio", meaning "reckoning" or "reasoning"
              $endgroup$
              – David Richerby
              Mar 8 at 20:12












            • 7




              $begingroup$
              Wikipedia calls the quaternions a number system.
              $endgroup$
              – JJJ
              Mar 8 at 4:52






            • 6




              $begingroup$
              You can also tell what opinion people had of a number system based on its name, generally (it is not a coincidence that the word rational means both a number system and a way of thinking). At least the old ones.
              $endgroup$
              – PyRulez
              Mar 8 at 6:26






            • 1




              $begingroup$
              Agreed; the use of the word number is definitely a matter of convention. As for opinions; the use of the word 'real' numbers is regarded as an incredibly triggering piece of propaganda by physical finitists...
              $endgroup$
              – Eelco Hoogendoorn
              Mar 8 at 12:40






            • 2




              $begingroup$
              @PyRulez I would have expected that "rational" comes from "ratio", not from "ration".
              $endgroup$
              – Thern
              Mar 8 at 16:31






            • 2




              $begingroup$
              @Thern The English words "ratio", "rational" and "reason" all come from the same Latin root "ratio", meaning "reckoning" or "reasoning"
              $endgroup$
              – David Richerby
              Mar 8 at 20:12







            7




            7




            $begingroup$
            Wikipedia calls the quaternions a number system.
            $endgroup$
            – JJJ
            Mar 8 at 4:52




            $begingroup$
            Wikipedia calls the quaternions a number system.
            $endgroup$
            – JJJ
            Mar 8 at 4:52




            6




            6




            $begingroup$
            You can also tell what opinion people had of a number system based on its name, generally (it is not a coincidence that the word rational means both a number system and a way of thinking). At least the old ones.
            $endgroup$
            – PyRulez
            Mar 8 at 6:26




            $begingroup$
            You can also tell what opinion people had of a number system based on its name, generally (it is not a coincidence that the word rational means both a number system and a way of thinking). At least the old ones.
            $endgroup$
            – PyRulez
            Mar 8 at 6:26




            1




            1




            $begingroup$
            Agreed; the use of the word number is definitely a matter of convention. As for opinions; the use of the word 'real' numbers is regarded as an incredibly triggering piece of propaganda by physical finitists...
            $endgroup$
            – Eelco Hoogendoorn
            Mar 8 at 12:40




            $begingroup$
            Agreed; the use of the word number is definitely a matter of convention. As for opinions; the use of the word 'real' numbers is regarded as an incredibly triggering piece of propaganda by physical finitists...
            $endgroup$
            – Eelco Hoogendoorn
            Mar 8 at 12:40




            2




            2




            $begingroup$
            @PyRulez I would have expected that "rational" comes from "ratio", not from "ration".
            $endgroup$
            – Thern
            Mar 8 at 16:31




            $begingroup$
            @PyRulez I would have expected that "rational" comes from "ratio", not from "ration".
            $endgroup$
            – Thern
            Mar 8 at 16:31




            2




            2




            $begingroup$
            @Thern The English words "ratio", "rational" and "reason" all come from the same Latin root "ratio", meaning "reckoning" or "reasoning"
            $endgroup$
            – David Richerby
            Mar 8 at 20:12




            $begingroup$
            @Thern The English words "ratio", "rational" and "reason" all come from the same Latin root "ratio", meaning "reckoning" or "reasoning"
            $endgroup$
            – David Richerby
            Mar 8 at 20:12











            20












            $begingroup$

            The two fundamental operations for numbers are "addition" and "multiplication" which obey very nice "laws" of arithmetic. Taking powers is also important. You can do all of those things with complex numbers. You can add two vectors but the "dot" product of two vectors is not a vector and the "cross" product of two vectors does not satisfy the "nice laws". Neither the dot product nor the cross product of vectors can be used to define powers.






            share|cite|improve this answer









            $endgroup$












            • $begingroup$
              So Quaternions which have Non commutativity multiplication would also be considered as non numbers?
              $endgroup$
              – Q the Platypus
              Mar 7 at 23:34






            • 5




              $begingroup$
              That's a good point- though I have never liked quaternions! Multiplication of quaternions is not commutative but it is associative which allows powers. That's what I was really thinking of.
              $endgroup$
              – user247327
              Mar 7 at 23:42






            • 4




              $begingroup$
              You can also add and multiply matrices, but I don't think I've ever heard anyone call a matrix a "number".
              $endgroup$
              – Hong Ooi
              Mar 8 at 2:48






            • 8




              $begingroup$
              @user247327: It's worth pointing out that IEEE 754 floating point numbers are non-associative, closed under division, non-reflexive (i.e. they violate $forall a:a = a$), non-substitutable (i.e. numbers which are "equal" cannot always be substituted for one another in expressions or equations), and also the most widely used approximation of the reals in practical computing applications.
              $endgroup$
              – Kevin
              Mar 8 at 2:48







            • 4




              $begingroup$
              @Todd Sewell - I think this may relate to positive and negative zero. Positive and negative zero are equal, yet if you divide something by one or the other you may get positive or negative infinity.
              $endgroup$
              – Hammerite
              Mar 8 at 13:27















            20












            $begingroup$

            The two fundamental operations for numbers are "addition" and "multiplication" which obey very nice "laws" of arithmetic. Taking powers is also important. You can do all of those things with complex numbers. You can add two vectors but the "dot" product of two vectors is not a vector and the "cross" product of two vectors does not satisfy the "nice laws". Neither the dot product nor the cross product of vectors can be used to define powers.






            share|cite|improve this answer









            $endgroup$












            • $begingroup$
              So Quaternions which have Non commutativity multiplication would also be considered as non numbers?
              $endgroup$
              – Q the Platypus
              Mar 7 at 23:34






            • 5




              $begingroup$
              That's a good point- though I have never liked quaternions! Multiplication of quaternions is not commutative but it is associative which allows powers. That's what I was really thinking of.
              $endgroup$
              – user247327
              Mar 7 at 23:42






            • 4




              $begingroup$
              You can also add and multiply matrices, but I don't think I've ever heard anyone call a matrix a "number".
              $endgroup$
              – Hong Ooi
              Mar 8 at 2:48






            • 8




              $begingroup$
              @user247327: It's worth pointing out that IEEE 754 floating point numbers are non-associative, closed under division, non-reflexive (i.e. they violate $forall a:a = a$), non-substitutable (i.e. numbers which are "equal" cannot always be substituted for one another in expressions or equations), and also the most widely used approximation of the reals in practical computing applications.
              $endgroup$
              – Kevin
              Mar 8 at 2:48







            • 4




              $begingroup$
              @Todd Sewell - I think this may relate to positive and negative zero. Positive and negative zero are equal, yet if you divide something by one or the other you may get positive or negative infinity.
              $endgroup$
              – Hammerite
              Mar 8 at 13:27













            20












            20








            20





            $begingroup$

            The two fundamental operations for numbers are "addition" and "multiplication" which obey very nice "laws" of arithmetic. Taking powers is also important. You can do all of those things with complex numbers. You can add two vectors but the "dot" product of two vectors is not a vector and the "cross" product of two vectors does not satisfy the "nice laws". Neither the dot product nor the cross product of vectors can be used to define powers.






            share|cite|improve this answer









            $endgroup$



            The two fundamental operations for numbers are "addition" and "multiplication" which obey very nice "laws" of arithmetic. Taking powers is also important. You can do all of those things with complex numbers. You can add two vectors but the "dot" product of two vectors is not a vector and the "cross" product of two vectors does not satisfy the "nice laws". Neither the dot product nor the cross product of vectors can be used to define powers.







            share|cite|improve this answer












            share|cite|improve this answer



            share|cite|improve this answer










            answered Mar 7 at 23:29









            user247327user247327

            11.5k1516




            11.5k1516











            • $begingroup$
              So Quaternions which have Non commutativity multiplication would also be considered as non numbers?
              $endgroup$
              – Q the Platypus
              Mar 7 at 23:34






            • 5




              $begingroup$
              That's a good point- though I have never liked quaternions! Multiplication of quaternions is not commutative but it is associative which allows powers. That's what I was really thinking of.
              $endgroup$
              – user247327
              Mar 7 at 23:42






            • 4




              $begingroup$
              You can also add and multiply matrices, but I don't think I've ever heard anyone call a matrix a "number".
              $endgroup$
              – Hong Ooi
              Mar 8 at 2:48






            • 8




              $begingroup$
              @user247327: It's worth pointing out that IEEE 754 floating point numbers are non-associative, closed under division, non-reflexive (i.e. they violate $forall a:a = a$), non-substitutable (i.e. numbers which are "equal" cannot always be substituted for one another in expressions or equations), and also the most widely used approximation of the reals in practical computing applications.
              $endgroup$
              – Kevin
              Mar 8 at 2:48







            • 4




              $begingroup$
              @Todd Sewell - I think this may relate to positive and negative zero. Positive and negative zero are equal, yet if you divide something by one or the other you may get positive or negative infinity.
              $endgroup$
              – Hammerite
              Mar 8 at 13:27
















            • $begingroup$
              So Quaternions which have Non commutativity multiplication would also be considered as non numbers?
              $endgroup$
              – Q the Platypus
              Mar 7 at 23:34






            • 5




              $begingroup$
              That's a good point- though I have never liked quaternions! Multiplication of quaternions is not commutative but it is associative which allows powers. That's what I was really thinking of.
              $endgroup$
              – user247327
              Mar 7 at 23:42






            • 4




              $begingroup$
              You can also add and multiply matrices, but I don't think I've ever heard anyone call a matrix a "number".
              $endgroup$
              – Hong Ooi
              Mar 8 at 2:48






            • 8




              $begingroup$
              @user247327: It's worth pointing out that IEEE 754 floating point numbers are non-associative, closed under division, non-reflexive (i.e. they violate $forall a:a = a$), non-substitutable (i.e. numbers which are "equal" cannot always be substituted for one another in expressions or equations), and also the most widely used approximation of the reals in practical computing applications.
              $endgroup$
              – Kevin
              Mar 8 at 2:48







            • 4




              $begingroup$
              @Todd Sewell - I think this may relate to positive and negative zero. Positive and negative zero are equal, yet if you divide something by one or the other you may get positive or negative infinity.
              $endgroup$
              – Hammerite
              Mar 8 at 13:27















            $begingroup$
            So Quaternions which have Non commutativity multiplication would also be considered as non numbers?
            $endgroup$
            – Q the Platypus
            Mar 7 at 23:34




            $begingroup$
            So Quaternions which have Non commutativity multiplication would also be considered as non numbers?
            $endgroup$
            – Q the Platypus
            Mar 7 at 23:34




            5




            5




            $begingroup$
            That's a good point- though I have never liked quaternions! Multiplication of quaternions is not commutative but it is associative which allows powers. That's what I was really thinking of.
            $endgroup$
            – user247327
            Mar 7 at 23:42




            $begingroup$
            That's a good point- though I have never liked quaternions! Multiplication of quaternions is not commutative but it is associative which allows powers. That's what I was really thinking of.
            $endgroup$
            – user247327
            Mar 7 at 23:42




            4




            4




            $begingroup$
            You can also add and multiply matrices, but I don't think I've ever heard anyone call a matrix a "number".
            $endgroup$
            – Hong Ooi
            Mar 8 at 2:48




            $begingroup$
            You can also add and multiply matrices, but I don't think I've ever heard anyone call a matrix a "number".
            $endgroup$
            – Hong Ooi
            Mar 8 at 2:48




            8




            8




            $begingroup$
            @user247327: It's worth pointing out that IEEE 754 floating point numbers are non-associative, closed under division, non-reflexive (i.e. they violate $forall a:a = a$), non-substitutable (i.e. numbers which are "equal" cannot always be substituted for one another in expressions or equations), and also the most widely used approximation of the reals in practical computing applications.
            $endgroup$
            – Kevin
            Mar 8 at 2:48





            $begingroup$
            @user247327: It's worth pointing out that IEEE 754 floating point numbers are non-associative, closed under division, non-reflexive (i.e. they violate $forall a:a = a$), non-substitutable (i.e. numbers which are "equal" cannot always be substituted for one another in expressions or equations), and also the most widely used approximation of the reals in practical computing applications.
            $endgroup$
            – Kevin
            Mar 8 at 2:48





            4




            4




            $begingroup$
            @Todd Sewell - I think this may relate to positive and negative zero. Positive and negative zero are equal, yet if you divide something by one or the other you may get positive or negative infinity.
            $endgroup$
            – Hammerite
            Mar 8 at 13:27




            $begingroup$
            @Todd Sewell - I think this may relate to positive and negative zero. Positive and negative zero are equal, yet if you divide something by one or the other you may get positive or negative infinity.
            $endgroup$
            – Hammerite
            Mar 8 at 13:27











            4












            $begingroup$

            Numbers appeared first when we started to count things. $1$ tree, $2$ trees, $3$ trees, and so forth. That make up the set of non-zero natural numbers: $mathbbN$. Afterwards, people started to "count backwards" to get $mathbbZ$ (I'm just kidding, you can do some research on how negative numbers appeared historically). With the urge to divide things without remainders, the set of rational numbers $mathbbQ$ made its way to the world. A certain idea of geometric continuity gives us $mathbbR$. Finally we want all equations to have a root, that's how $mathbbC$ comes into play.



            I guess what is considered "numbers" is rather a social question. The use of $mathbbC$
            in physics and its $2$-D representation must have given a good intuition for a large set of people to accept it's intuitive enough to be considered "numbers".



            I think it's not that natural to think of $mathbbC$ as a $mathbbR$-vector space of dimension $2$, not more natural than to think of $mathbbR$ as an infinite-dimensional $mathbbQ$-vector space, nor of $mathbbQ$ as a non-infinitely-generated $mathbbZ$-module.






            share|cite|improve this answer











            $endgroup$








            • 3




              $begingroup$
              People do think of complex numbers as points on the number plane.
              $endgroup$
              – Q the Platypus
              Mar 7 at 23:48






            • 1




              $begingroup$
              Thank you. I'm sorry, that final part is just a personal opinion. edited.
              $endgroup$
              – Leaning
              Mar 7 at 23:48
















            4












            $begingroup$

            Numbers appeared first when we started to count things. $1$ tree, $2$ trees, $3$ trees, and so forth. That make up the set of non-zero natural numbers: $mathbbN$. Afterwards, people started to "count backwards" to get $mathbbZ$ (I'm just kidding, you can do some research on how negative numbers appeared historically). With the urge to divide things without remainders, the set of rational numbers $mathbbQ$ made its way to the world. A certain idea of geometric continuity gives us $mathbbR$. Finally we want all equations to have a root, that's how $mathbbC$ comes into play.



            I guess what is considered "numbers" is rather a social question. The use of $mathbbC$
            in physics and its $2$-D representation must have given a good intuition for a large set of people to accept it's intuitive enough to be considered "numbers".



            I think it's not that natural to think of $mathbbC$ as a $mathbbR$-vector space of dimension $2$, not more natural than to think of $mathbbR$ as an infinite-dimensional $mathbbQ$-vector space, nor of $mathbbQ$ as a non-infinitely-generated $mathbbZ$-module.






            share|cite|improve this answer











            $endgroup$








            • 3




              $begingroup$
              People do think of complex numbers as points on the number plane.
              $endgroup$
              – Q the Platypus
              Mar 7 at 23:48






            • 1




              $begingroup$
              Thank you. I'm sorry, that final part is just a personal opinion. edited.
              $endgroup$
              – Leaning
              Mar 7 at 23:48














            4












            4








            4





            $begingroup$

            Numbers appeared first when we started to count things. $1$ tree, $2$ trees, $3$ trees, and so forth. That make up the set of non-zero natural numbers: $mathbbN$. Afterwards, people started to "count backwards" to get $mathbbZ$ (I'm just kidding, you can do some research on how negative numbers appeared historically). With the urge to divide things without remainders, the set of rational numbers $mathbbQ$ made its way to the world. A certain idea of geometric continuity gives us $mathbbR$. Finally we want all equations to have a root, that's how $mathbbC$ comes into play.



            I guess what is considered "numbers" is rather a social question. The use of $mathbbC$
            in physics and its $2$-D representation must have given a good intuition for a large set of people to accept it's intuitive enough to be considered "numbers".



            I think it's not that natural to think of $mathbbC$ as a $mathbbR$-vector space of dimension $2$, not more natural than to think of $mathbbR$ as an infinite-dimensional $mathbbQ$-vector space, nor of $mathbbQ$ as a non-infinitely-generated $mathbbZ$-module.






            share|cite|improve this answer











            $endgroup$



            Numbers appeared first when we started to count things. $1$ tree, $2$ trees, $3$ trees, and so forth. That make up the set of non-zero natural numbers: $mathbbN$. Afterwards, people started to "count backwards" to get $mathbbZ$ (I'm just kidding, you can do some research on how negative numbers appeared historically). With the urge to divide things without remainders, the set of rational numbers $mathbbQ$ made its way to the world. A certain idea of geometric continuity gives us $mathbbR$. Finally we want all equations to have a root, that's how $mathbbC$ comes into play.



            I guess what is considered "numbers" is rather a social question. The use of $mathbbC$
            in physics and its $2$-D representation must have given a good intuition for a large set of people to accept it's intuitive enough to be considered "numbers".



            I think it's not that natural to think of $mathbbC$ as a $mathbbR$-vector space of dimension $2$, not more natural than to think of $mathbbR$ as an infinite-dimensional $mathbbQ$-vector space, nor of $mathbbQ$ as a non-infinitely-generated $mathbbZ$-module.







            share|cite|improve this answer














            share|cite|improve this answer



            share|cite|improve this answer








            edited Mar 7 at 23:49

























            answered Mar 7 at 23:45









            LeaningLeaning

            1,331718




            1,331718







            • 3




              $begingroup$
              People do think of complex numbers as points on the number plane.
              $endgroup$
              – Q the Platypus
              Mar 7 at 23:48






            • 1




              $begingroup$
              Thank you. I'm sorry, that final part is just a personal opinion. edited.
              $endgroup$
              – Leaning
              Mar 7 at 23:48













            • 3




              $begingroup$
              People do think of complex numbers as points on the number plane.
              $endgroup$
              – Q the Platypus
              Mar 7 at 23:48






            • 1




              $begingroup$
              Thank you. I'm sorry, that final part is just a personal opinion. edited.
              $endgroup$
              – Leaning
              Mar 7 at 23:48








            3




            3




            $begingroup$
            People do think of complex numbers as points on the number plane.
            $endgroup$
            – Q the Platypus
            Mar 7 at 23:48




            $begingroup$
            People do think of complex numbers as points on the number plane.
            $endgroup$
            – Q the Platypus
            Mar 7 at 23:48




            1




            1




            $begingroup$
            Thank you. I'm sorry, that final part is just a personal opinion. edited.
            $endgroup$
            – Leaning
            Mar 7 at 23:48





            $begingroup$
            Thank you. I'm sorry, that final part is just a personal opinion. edited.
            $endgroup$
            – Leaning
            Mar 7 at 23:48












            3












            $begingroup$

            I think naming is a rather soft topic, so there may not be a hard answer. However, I think it is worth noting that complex numbers are one of the three associative real division algebras (real numbers, complex numbers, and quaternions). These are all linked by the idea that division is meaningful in those three systems.



            In general, a 2 dimensional real vector cannot admit a concept of division which matches what we expect division to do. However, if said real vector defines addition and multiplication the way complex numbers do, division is a natural result of those definitions.






            share|cite|improve this answer









            $endgroup$

















              3












              $begingroup$

              I think naming is a rather soft topic, so there may not be a hard answer. However, I think it is worth noting that complex numbers are one of the three associative real division algebras (real numbers, complex numbers, and quaternions). These are all linked by the idea that division is meaningful in those three systems.



              In general, a 2 dimensional real vector cannot admit a concept of division which matches what we expect division to do. However, if said real vector defines addition and multiplication the way complex numbers do, division is a natural result of those definitions.






              share|cite|improve this answer









              $endgroup$















                3












                3








                3





                $begingroup$

                I think naming is a rather soft topic, so there may not be a hard answer. However, I think it is worth noting that complex numbers are one of the three associative real division algebras (real numbers, complex numbers, and quaternions). These are all linked by the idea that division is meaningful in those three systems.



                In general, a 2 dimensional real vector cannot admit a concept of division which matches what we expect division to do. However, if said real vector defines addition and multiplication the way complex numbers do, division is a natural result of those definitions.






                share|cite|improve this answer









                $endgroup$



                I think naming is a rather soft topic, so there may not be a hard answer. However, I think it is worth noting that complex numbers are one of the three associative real division algebras (real numbers, complex numbers, and quaternions). These are all linked by the idea that division is meaningful in those three systems.



                In general, a 2 dimensional real vector cannot admit a concept of division which matches what we expect division to do. However, if said real vector defines addition and multiplication the way complex numbers do, division is a natural result of those definitions.







                share|cite|improve this answer












                share|cite|improve this answer



                share|cite|improve this answer










                answered Mar 8 at 4:45









                Cort AmmonCort Ammon

                2,451716




                2,451716





















                    2












                    $begingroup$

                    Both, complex numbers and vector spaces are mathematical spaces, i.e., sets equipped it with some structure, e.g., operations (such as addition or multiplication) and rules obeyed by them (such as commutativity or associativity).
                    Without that structure, these sets are pretty useless:
                    You can hardly make any interesting statements about them or connect them to reality.



                    For example, without any structure, the only difference between ℝ and ℝ² is how we tend to name the objects within: We can find bijective maps between them.
                    Of course, once we impose the usual structure onto these sets, we will note that these maps do not preserve any of it (they are not even continuous) and thus are not very relevant.



                    Now, for ℝ² (the space of 2-vectors) and ℂ, some structure such as addition is the same, but some isn’t, e.g., there is no equivalent to complex multiplication in ℝ².
                    This distinction in structure is the only relevant difference between ℝ² and ℂ.
                    If you define a multiplication of 2-vectors that just mirrors complex multiplication, you would just be inventing complex numbers under a different name.



                    So much for why it is justified to apply different labels to complex numbers and 2-vectors.
                    As for why we call complex numbers numbers in the first place, there’s certainly a bit of history involved (of which I know little) and number is not a mathematically defined concept anyway.
                    That being said, calling complex numbers numbers is not completely deviod of consistency, given that complex numbers feature most of the structural properties of other sets that we call numbers (for even more historical reasons), e.g., the vast majority of things you can do with real numbers, you can also do with complex numbers.
                    This does not hold for 2-vectors.






                    share|cite|improve this answer









                    $endgroup$

















                      2












                      $begingroup$

                      Both, complex numbers and vector spaces are mathematical spaces, i.e., sets equipped it with some structure, e.g., operations (such as addition or multiplication) and rules obeyed by them (such as commutativity or associativity).
                      Without that structure, these sets are pretty useless:
                      You can hardly make any interesting statements about them or connect them to reality.



                      For example, without any structure, the only difference between ℝ and ℝ² is how we tend to name the objects within: We can find bijective maps between them.
                      Of course, once we impose the usual structure onto these sets, we will note that these maps do not preserve any of it (they are not even continuous) and thus are not very relevant.



                      Now, for ℝ² (the space of 2-vectors) and ℂ, some structure such as addition is the same, but some isn’t, e.g., there is no equivalent to complex multiplication in ℝ².
                      This distinction in structure is the only relevant difference between ℝ² and ℂ.
                      If you define a multiplication of 2-vectors that just mirrors complex multiplication, you would just be inventing complex numbers under a different name.



                      So much for why it is justified to apply different labels to complex numbers and 2-vectors.
                      As for why we call complex numbers numbers in the first place, there’s certainly a bit of history involved (of which I know little) and number is not a mathematically defined concept anyway.
                      That being said, calling complex numbers numbers is not completely deviod of consistency, given that complex numbers feature most of the structural properties of other sets that we call numbers (for even more historical reasons), e.g., the vast majority of things you can do with real numbers, you can also do with complex numbers.
                      This does not hold for 2-vectors.






                      share|cite|improve this answer









                      $endgroup$















                        2












                        2








                        2





                        $begingroup$

                        Both, complex numbers and vector spaces are mathematical spaces, i.e., sets equipped it with some structure, e.g., operations (such as addition or multiplication) and rules obeyed by them (such as commutativity or associativity).
                        Without that structure, these sets are pretty useless:
                        You can hardly make any interesting statements about them or connect them to reality.



                        For example, without any structure, the only difference between ℝ and ℝ² is how we tend to name the objects within: We can find bijective maps between them.
                        Of course, once we impose the usual structure onto these sets, we will note that these maps do not preserve any of it (they are not even continuous) and thus are not very relevant.



                        Now, for ℝ² (the space of 2-vectors) and ℂ, some structure such as addition is the same, but some isn’t, e.g., there is no equivalent to complex multiplication in ℝ².
                        This distinction in structure is the only relevant difference between ℝ² and ℂ.
                        If you define a multiplication of 2-vectors that just mirrors complex multiplication, you would just be inventing complex numbers under a different name.



                        So much for why it is justified to apply different labels to complex numbers and 2-vectors.
                        As for why we call complex numbers numbers in the first place, there’s certainly a bit of history involved (of which I know little) and number is not a mathematically defined concept anyway.
                        That being said, calling complex numbers numbers is not completely deviod of consistency, given that complex numbers feature most of the structural properties of other sets that we call numbers (for even more historical reasons), e.g., the vast majority of things you can do with real numbers, you can also do with complex numbers.
                        This does not hold for 2-vectors.






                        share|cite|improve this answer









                        $endgroup$



                        Both, complex numbers and vector spaces are mathematical spaces, i.e., sets equipped it with some structure, e.g., operations (such as addition or multiplication) and rules obeyed by them (such as commutativity or associativity).
                        Without that structure, these sets are pretty useless:
                        You can hardly make any interesting statements about them or connect them to reality.



                        For example, without any structure, the only difference between ℝ and ℝ² is how we tend to name the objects within: We can find bijective maps between them.
                        Of course, once we impose the usual structure onto these sets, we will note that these maps do not preserve any of it (they are not even continuous) and thus are not very relevant.



                        Now, for ℝ² (the space of 2-vectors) and ℂ, some structure such as addition is the same, but some isn’t, e.g., there is no equivalent to complex multiplication in ℝ².
                        This distinction in structure is the only relevant difference between ℝ² and ℂ.
                        If you define a multiplication of 2-vectors that just mirrors complex multiplication, you would just be inventing complex numbers under a different name.



                        So much for why it is justified to apply different labels to complex numbers and 2-vectors.
                        As for why we call complex numbers numbers in the first place, there’s certainly a bit of history involved (of which I know little) and number is not a mathematically defined concept anyway.
                        That being said, calling complex numbers numbers is not completely deviod of consistency, given that complex numbers feature most of the structural properties of other sets that we call numbers (for even more historical reasons), e.g., the vast majority of things you can do with real numbers, you can also do with complex numbers.
                        This does not hold for 2-vectors.







                        share|cite|improve this answer












                        share|cite|improve this answer



                        share|cite|improve this answer










                        answered Mar 8 at 7:41









                        WrzlprmftWrzlprmft

                        3,15111335




                        3,15111335





















                            1












                            $begingroup$

                            The answer by @MathematicsStudent1122 is good but doesn't address the question about vectors. The reason for using a distinctive name for such objects is that, in general, they do not admit all the arithmetical operations that have we come to expect with numbers. In particular, there is generally no natural or canonical way of multiplying or dividing vectors. Exceptionally, for two, three, or four dimensions, operations that correspond to multiplication or division of vectors do exist, although they are no longer commutative for dimensions 3 or 4; moreover, in three dimensions, vector multiplication has un-numberlike properties (e.g. $pmb xtimespmb x=pmb0$), and there is no natural way to define division. As you point out, real 2-D vectors can be identified with complex numbers. However, even in low-dimensional cases, typically vectors are not used in a context where any vector multiplication that could be defined has meaning.






                            share|cite|improve this answer









                            $endgroup$

















                              1












                              $begingroup$

                              The answer by @MathematicsStudent1122 is good but doesn't address the question about vectors. The reason for using a distinctive name for such objects is that, in general, they do not admit all the arithmetical operations that have we come to expect with numbers. In particular, there is generally no natural or canonical way of multiplying or dividing vectors. Exceptionally, for two, three, or four dimensions, operations that correspond to multiplication or division of vectors do exist, although they are no longer commutative for dimensions 3 or 4; moreover, in three dimensions, vector multiplication has un-numberlike properties (e.g. $pmb xtimespmb x=pmb0$), and there is no natural way to define division. As you point out, real 2-D vectors can be identified with complex numbers. However, even in low-dimensional cases, typically vectors are not used in a context where any vector multiplication that could be defined has meaning.






                              share|cite|improve this answer









                              $endgroup$















                                1












                                1








                                1





                                $begingroup$

                                The answer by @MathematicsStudent1122 is good but doesn't address the question about vectors. The reason for using a distinctive name for such objects is that, in general, they do not admit all the arithmetical operations that have we come to expect with numbers. In particular, there is generally no natural or canonical way of multiplying or dividing vectors. Exceptionally, for two, three, or four dimensions, operations that correspond to multiplication or division of vectors do exist, although they are no longer commutative for dimensions 3 or 4; moreover, in three dimensions, vector multiplication has un-numberlike properties (e.g. $pmb xtimespmb x=pmb0$), and there is no natural way to define division. As you point out, real 2-D vectors can be identified with complex numbers. However, even in low-dimensional cases, typically vectors are not used in a context where any vector multiplication that could be defined has meaning.






                                share|cite|improve this answer









                                $endgroup$



                                The answer by @MathematicsStudent1122 is good but doesn't address the question about vectors. The reason for using a distinctive name for such objects is that, in general, they do not admit all the arithmetical operations that have we come to expect with numbers. In particular, there is generally no natural or canonical way of multiplying or dividing vectors. Exceptionally, for two, three, or four dimensions, operations that correspond to multiplication or division of vectors do exist, although they are no longer commutative for dimensions 3 or 4; moreover, in three dimensions, vector multiplication has un-numberlike properties (e.g. $pmb xtimespmb x=pmb0$), and there is no natural way to define division. As you point out, real 2-D vectors can be identified with complex numbers. However, even in low-dimensional cases, typically vectors are not used in a context where any vector multiplication that could be defined has meaning.







                                share|cite|improve this answer












                                share|cite|improve this answer



                                share|cite|improve this answer










                                answered Mar 9 at 10:45









                                John BentinJohn Bentin

                                11.4k22554




                                11.4k22554





















                                    1












                                    $begingroup$

                                    In my observation, the term number is used almost exclusively in the following cases:



                                    • The natural numbers, basically the origin of the term.



                                    • Any system which extends a system already called “numbers” by adding numbers that are in some sense “missing” That is, there are some constructions which sometimes give a number, but sometimes don't, although is “looks like” such a number should exist.



                                      The actual construction of the extension is then different from just adding those “missing” numbers just in order to make sure that what you do makes sense. However the goal is always to add the “missing” numbers, and any further numbers that are implied by their existence, but nothing else.



                                    The classic sequence of number sets is exactly of this type:



                                    • The natural numbers allow to solve equations like $2+x=6$, but not equations like $5+x=2$. Adding solutions to the latter gives the integers (which, of course, are considered to be numbers).


                                    • The integers allow to solve equations like $2x=6$, but not equations like $5x=2$. Adding solutions to the latter gives the rational numbers.


                                    • In the rational numbers, all convergent sequences are Cauchy, but not all Cauchy sequences converge. But they look like they should converge. Thus the missing limits are added. (Note that there are other constructions of the real numbers, based on other constructions where numbers are found "missing").


                                    • In the real numbers, equations like $x^2=2$ can be solved, but equations like $x^2=-2$ cannot. Adding those numbers then gives the complex numbers.


                                    But is also is true for most other systems which are generally called numbers. For example:



                                    • Finite sets always have an integer number of elements. For infinite sets, there's no natural number describing the set's size. The cardinal numbers add those numbers describing the size of infinite sets.


                                    • Similarly, when considering well-orderings, you need a generalization of the terms “first”, “second”, … to infinite size orderings. The ordinal numbers provide those missing numbers.


                                    • When informally talking about differentiation and integration, one often uses the concept of “infinitesimal quantities”. Those don't exist in the real or complex numbers. Non-standard analysis adds those “missing” infinitesimals (and everything implied by them), arriving at the hyperreal numbers.


                                    • When looking at the sequence $r_n$ of remainders modulo $p^n$, $p$ prime, every integer gives an unique sequence. But not every sequence gives a number. The $p$-adic numbers add those “missing” numbers.


                                    Indeed, off the top of my head, I can only think of two constructions called “numbers” that fall outside this scheme:



                                    • The quaternions are not constructed as extension, but explicitly as “numbers” that allow to describe three-dimensional geometry, after seeing that complex numbers describe two-dimensional geometry. It turned out that you need a fourth dimension to get something meaningful. However not everyone considers the quaternions to be numbers.


                                    • The surreal numbers are not constructed as extension to anything, but arise independently from combinatoric game theory. However since not only behave very much like numbers do, but in addition contain subsets isomorphic to several other number sets or classes (you'll find subsets isomorphic to the real numbers, to all systems of hyperreal numbers, and to the ordinal numbers under natural arithmetic). So while the surreal numbers are not constructed as extension, they can be viewed in some sense as such, justifying the term “number”.






                                    share|cite|improve this answer











                                    $endgroup$

















                                      1












                                      $begingroup$

                                      In my observation, the term number is used almost exclusively in the following cases:



                                      • The natural numbers, basically the origin of the term.



                                      • Any system which extends a system already called “numbers” by adding numbers that are in some sense “missing” That is, there are some constructions which sometimes give a number, but sometimes don't, although is “looks like” such a number should exist.



                                        The actual construction of the extension is then different from just adding those “missing” numbers just in order to make sure that what you do makes sense. However the goal is always to add the “missing” numbers, and any further numbers that are implied by their existence, but nothing else.



                                      The classic sequence of number sets is exactly of this type:



                                      • The natural numbers allow to solve equations like $2+x=6$, but not equations like $5+x=2$. Adding solutions to the latter gives the integers (which, of course, are considered to be numbers).


                                      • The integers allow to solve equations like $2x=6$, but not equations like $5x=2$. Adding solutions to the latter gives the rational numbers.


                                      • In the rational numbers, all convergent sequences are Cauchy, but not all Cauchy sequences converge. But they look like they should converge. Thus the missing limits are added. (Note that there are other constructions of the real numbers, based on other constructions where numbers are found "missing").


                                      • In the real numbers, equations like $x^2=2$ can be solved, but equations like $x^2=-2$ cannot. Adding those numbers then gives the complex numbers.


                                      But is also is true for most other systems which are generally called numbers. For example:



                                      • Finite sets always have an integer number of elements. For infinite sets, there's no natural number describing the set's size. The cardinal numbers add those numbers describing the size of infinite sets.


                                      • Similarly, when considering well-orderings, you need a generalization of the terms “first”, “second”, … to infinite size orderings. The ordinal numbers provide those missing numbers.


                                      • When informally talking about differentiation and integration, one often uses the concept of “infinitesimal quantities”. Those don't exist in the real or complex numbers. Non-standard analysis adds those “missing” infinitesimals (and everything implied by them), arriving at the hyperreal numbers.


                                      • When looking at the sequence $r_n$ of remainders modulo $p^n$, $p$ prime, every integer gives an unique sequence. But not every sequence gives a number. The $p$-adic numbers add those “missing” numbers.


                                      Indeed, off the top of my head, I can only think of two constructions called “numbers” that fall outside this scheme:



                                      • The quaternions are not constructed as extension, but explicitly as “numbers” that allow to describe three-dimensional geometry, after seeing that complex numbers describe two-dimensional geometry. It turned out that you need a fourth dimension to get something meaningful. However not everyone considers the quaternions to be numbers.


                                      • The surreal numbers are not constructed as extension to anything, but arise independently from combinatoric game theory. However since not only behave very much like numbers do, but in addition contain subsets isomorphic to several other number sets or classes (you'll find subsets isomorphic to the real numbers, to all systems of hyperreal numbers, and to the ordinal numbers under natural arithmetic). So while the surreal numbers are not constructed as extension, they can be viewed in some sense as such, justifying the term “number”.






                                      share|cite|improve this answer











                                      $endgroup$















                                        1












                                        1








                                        1





                                        $begingroup$

                                        In my observation, the term number is used almost exclusively in the following cases:



                                        • The natural numbers, basically the origin of the term.



                                        • Any system which extends a system already called “numbers” by adding numbers that are in some sense “missing” That is, there are some constructions which sometimes give a number, but sometimes don't, although is “looks like” such a number should exist.



                                          The actual construction of the extension is then different from just adding those “missing” numbers just in order to make sure that what you do makes sense. However the goal is always to add the “missing” numbers, and any further numbers that are implied by their existence, but nothing else.



                                        The classic sequence of number sets is exactly of this type:



                                        • The natural numbers allow to solve equations like $2+x=6$, but not equations like $5+x=2$. Adding solutions to the latter gives the integers (which, of course, are considered to be numbers).


                                        • The integers allow to solve equations like $2x=6$, but not equations like $5x=2$. Adding solutions to the latter gives the rational numbers.


                                        • In the rational numbers, all convergent sequences are Cauchy, but not all Cauchy sequences converge. But they look like they should converge. Thus the missing limits are added. (Note that there are other constructions of the real numbers, based on other constructions where numbers are found "missing").


                                        • In the real numbers, equations like $x^2=2$ can be solved, but equations like $x^2=-2$ cannot. Adding those numbers then gives the complex numbers.


                                        But is also is true for most other systems which are generally called numbers. For example:



                                        • Finite sets always have an integer number of elements. For infinite sets, there's no natural number describing the set's size. The cardinal numbers add those numbers describing the size of infinite sets.


                                        • Similarly, when considering well-orderings, you need a generalization of the terms “first”, “second”, … to infinite size orderings. The ordinal numbers provide those missing numbers.


                                        • When informally talking about differentiation and integration, one often uses the concept of “infinitesimal quantities”. Those don't exist in the real or complex numbers. Non-standard analysis adds those “missing” infinitesimals (and everything implied by them), arriving at the hyperreal numbers.


                                        • When looking at the sequence $r_n$ of remainders modulo $p^n$, $p$ prime, every integer gives an unique sequence. But not every sequence gives a number. The $p$-adic numbers add those “missing” numbers.


                                        Indeed, off the top of my head, I can only think of two constructions called “numbers” that fall outside this scheme:



                                        • The quaternions are not constructed as extension, but explicitly as “numbers” that allow to describe three-dimensional geometry, after seeing that complex numbers describe two-dimensional geometry. It turned out that you need a fourth dimension to get something meaningful. However not everyone considers the quaternions to be numbers.


                                        • The surreal numbers are not constructed as extension to anything, but arise independently from combinatoric game theory. However since not only behave very much like numbers do, but in addition contain subsets isomorphic to several other number sets or classes (you'll find subsets isomorphic to the real numbers, to all systems of hyperreal numbers, and to the ordinal numbers under natural arithmetic). So while the surreal numbers are not constructed as extension, they can be viewed in some sense as such, justifying the term “number”.






                                        share|cite|improve this answer











                                        $endgroup$



                                        In my observation, the term number is used almost exclusively in the following cases:



                                        • The natural numbers, basically the origin of the term.



                                        • Any system which extends a system already called “numbers” by adding numbers that are in some sense “missing” That is, there are some constructions which sometimes give a number, but sometimes don't, although is “looks like” such a number should exist.



                                          The actual construction of the extension is then different from just adding those “missing” numbers just in order to make sure that what you do makes sense. However the goal is always to add the “missing” numbers, and any further numbers that are implied by their existence, but nothing else.



                                        The classic sequence of number sets is exactly of this type:



                                        • The natural numbers allow to solve equations like $2+x=6$, but not equations like $5+x=2$. Adding solutions to the latter gives the integers (which, of course, are considered to be numbers).


                                        • The integers allow to solve equations like $2x=6$, but not equations like $5x=2$. Adding solutions to the latter gives the rational numbers.


                                        • In the rational numbers, all convergent sequences are Cauchy, but not all Cauchy sequences converge. But they look like they should converge. Thus the missing limits are added. (Note that there are other constructions of the real numbers, based on other constructions where numbers are found "missing").


                                        • In the real numbers, equations like $x^2=2$ can be solved, but equations like $x^2=-2$ cannot. Adding those numbers then gives the complex numbers.


                                        But is also is true for most other systems which are generally called numbers. For example:



                                        • Finite sets always have an integer number of elements. For infinite sets, there's no natural number describing the set's size. The cardinal numbers add those numbers describing the size of infinite sets.


                                        • Similarly, when considering well-orderings, you need a generalization of the terms “first”, “second”, … to infinite size orderings. The ordinal numbers provide those missing numbers.


                                        • When informally talking about differentiation and integration, one often uses the concept of “infinitesimal quantities”. Those don't exist in the real or complex numbers. Non-standard analysis adds those “missing” infinitesimals (and everything implied by them), arriving at the hyperreal numbers.


                                        • When looking at the sequence $r_n$ of remainders modulo $p^n$, $p$ prime, every integer gives an unique sequence. But not every sequence gives a number. The $p$-adic numbers add those “missing” numbers.


                                        Indeed, off the top of my head, I can only think of two constructions called “numbers” that fall outside this scheme:



                                        • The quaternions are not constructed as extension, but explicitly as “numbers” that allow to describe three-dimensional geometry, after seeing that complex numbers describe two-dimensional geometry. It turned out that you need a fourth dimension to get something meaningful. However not everyone considers the quaternions to be numbers.


                                        • The surreal numbers are not constructed as extension to anything, but arise independently from combinatoric game theory. However since not only behave very much like numbers do, but in addition contain subsets isomorphic to several other number sets or classes (you'll find subsets isomorphic to the real numbers, to all systems of hyperreal numbers, and to the ordinal numbers under natural arithmetic). So while the surreal numbers are not constructed as extension, they can be viewed in some sense as such, justifying the term “number”.







                                        share|cite|improve this answer














                                        share|cite|improve this answer



                                        share|cite|improve this answer








                                        edited Mar 9 at 11:00

























                                        answered Mar 9 at 10:54









                                        celtschkceltschk

                                        30.3k755101




                                        30.3k755101





















                                            0












                                            $begingroup$

                                            [Possibly not a direct answer, but]



                                            Notice ordinal sum and product are not commutative, that full distributivity of multiplication over addition and of exponentiation over product fails, and they're still regarded as numbers. OTOH, the corresponding operations for vector spaces are well-behaved (probably because they do become cardinal-like once you identify the isomorphic ones) and yet we don't tend to think of or to refer to them as numbers.






                                            share|cite|improve this answer









                                            $endgroup$

















                                              0












                                              $begingroup$

                                              [Possibly not a direct answer, but]



                                              Notice ordinal sum and product are not commutative, that full distributivity of multiplication over addition and of exponentiation over product fails, and they're still regarded as numbers. OTOH, the corresponding operations for vector spaces are well-behaved (probably because they do become cardinal-like once you identify the isomorphic ones) and yet we don't tend to think of or to refer to them as numbers.






                                              share|cite|improve this answer









                                              $endgroup$















                                                0












                                                0








                                                0





                                                $begingroup$

                                                [Possibly not a direct answer, but]



                                                Notice ordinal sum and product are not commutative, that full distributivity of multiplication over addition and of exponentiation over product fails, and they're still regarded as numbers. OTOH, the corresponding operations for vector spaces are well-behaved (probably because they do become cardinal-like once you identify the isomorphic ones) and yet we don't tend to think of or to refer to them as numbers.






                                                share|cite|improve this answer









                                                $endgroup$



                                                [Possibly not a direct answer, but]



                                                Notice ordinal sum and product are not commutative, that full distributivity of multiplication over addition and of exponentiation over product fails, and they're still regarded as numbers. OTOH, the corresponding operations for vector spaces are well-behaved (probably because they do become cardinal-like once you identify the isomorphic ones) and yet we don't tend to think of or to refer to them as numbers.







                                                share|cite|improve this answer












                                                share|cite|improve this answer



                                                share|cite|improve this answer










                                                answered Mar 8 at 16:02









                                                alkchfalkchf

                                                47337




                                                47337



























                                                    draft saved

                                                    draft discarded
















































                                                    Thanks for contributing an answer to Mathematics Stack Exchange!


                                                    • Please be sure to answer the question. Provide details and share your research!

                                                    But avoid


                                                    • Asking for help, clarification, or responding to other answers.

                                                    • Making statements based on opinion; back them up with references or personal experience.

                                                    Use MathJax to format equations. MathJax reference.


                                                    To learn more, see our tips on writing great answers.




                                                    draft saved


                                                    draft discarded














                                                    StackExchange.ready(
                                                    function ()
                                                    StackExchange.openid.initPostLogin('.new-post-login', 'https%3a%2f%2fmath.stackexchange.com%2fquestions%2f3139486%2fwhy-do-we-call-complex-numbers-numbers-but-we-don-t-consider-2-vectors-numbers%23new-answer', 'question_page');

                                                    );

                                                    Post as a guest















                                                    Required, but never shown





















































                                                    Required, but never shown














                                                    Required, but never shown












                                                    Required, but never shown







                                                    Required, but never shown

































                                                    Required, but never shown














                                                    Required, but never shown












                                                    Required, but never shown







                                                    Required, but never shown







                                                    Popular posts from this blog

                                                    Can't initialize raids on a new ASUS Prime B360M-A motherboard2019 Community Moderator ElectionSimilar to RAID config yet more like mirroring solution?Can't get motherboard serial numberWhy does the BIOS entry point start with a WBINVD instruction?UEFI performance Asus Maximus V Extreme

                                                    Identity Server 4 is not redirecting to Angular app after login2019 Community Moderator ElectionIdentity Server 4 and dockerIdentityserver implicit flow unauthorized_clientIdentityServer Hybrid Flow - Access Token is null after user successful loginIdentity Server to MVC client : Page Redirect After loginLogin with Steam OpenId(oidc-client-js)Identity Server 4+.NET Core 2.0 + IdentityIdentityServer4 post-login redirect not working in Edge browserCall to IdentityServer4 generates System.NullReferenceException: Object reference not set to an instance of an objectIdentityServer4 without HTTPS not workingHow to get Authorization code from identity server without login form

                                                    2005 Ahvaz unrest Contents Background Causes Casualties Aftermath See also References Navigation menue"At Least 10 Are Killed by Bombs in Iran""Iran"Archived"Arab-Iranians in Iran to make April 15 'Day of Fury'"State of Mind, State of Order: Reactions to Ethnic Unrest in the Islamic Republic of Iran.10.1111/j.1754-9469.2008.00028.x"Iran hangs Arab separatists"Iran Overview from ArchivedConstitution of the Islamic Republic of Iran"Tehran puzzled by forged 'riots' letter""Iran and its minorities: Down in the second class""Iran: Handling Of Ahvaz Unrest Could End With Televised Confessions""Bombings Rock Iran Ahead of Election""Five die in Iran ethnic clashes""Iran: Need for restraint as anniversary of unrest in Khuzestan approaches"Archived"Iranian Sunni protesters killed in clashes with security forces"Archived